LSAT and Law School Admissions Forum

Get expert LSAT preparation and law school admissions advice from PowerScore Test Preparation.

 8scn
  • Posts: 18
  • Joined: Nov 21, 2011
|
#3047
Why is B not correct? For B, my reasoning is: having a broad range of knowledge is irrelevant to succeeding in this particular course. I eliminated A because shouldn't it increase the student's chances of succeeding with a B in the course if he is receiving extra help from smarter students?
 Steve Stein
PowerScore Staff
  • PowerScore Staff
  • Posts: 1153
  • Joined: Apr 11, 2011
|
#3052
Keep in mind that the four wrong answer choices will successfully strengthen the argument that the student is likely to get a B in the present course, much like she has in the majority of past cases.

If she is getting extra help, as answer choice A provides, this does not strengthen the argument--in fact, it seems to make it less likely that her current performance will be consistent with her past performances.

Answer choice B, on the other hand, does strengthen the author's argument, since previous broad subject coverage makes her past twelve performances a more accurate basis for predicting the grade she'll earn in the course that she's currently taking.

Tricky! let me know if that makes sense--thanks!

~Steve
 8scn
  • Posts: 18
  • Joined: Nov 21, 2011
|
#3104
Thanks very much, I understand now. I agree that it is quite tricky, especially for a first logical reasoning question.
User avatar
 queenbee
  • Posts: 75
  • Joined: Sep 18, 2022
|
#99120
Hi
Not sure I follow the reasoning. If the student is studying with other smart students, should that increase her chances of doing well? If the 12 previous courses she took were broad in nature, that doesnt guarantee that this course is broad in nature.
Any chance you can provide more clarity?
Thanks!
 Adam Tyson
PowerScore Staff
  • PowerScore Staff
  • Posts: 5153
  • Joined: Apr 14, 2011
|
#99241
You're right about answer A, queenbee - studying with a bunch of outstanding students rather than studying alone should improve her chances of getting a better grade, maybe an A this time. That's exactly why this is the correct answer! This is an "Except" question, so we want the one answer that will not strengthen the claim that she will probably get another B. Every other answer choice helps to support the claim that this course will turn out like the majority of other courses, so they all strengthen that claim and are, therefore, wrong answers. Answer A doesn't help that argument, and may in fact weaken it, so it's the exception we are looking for!

Get the most out of your LSAT Prep Plus subscription.

Analyze and track your performance with our Testing and Analytics Package.